Login

Welcome, Guest. Please login or register.

April 27, 2024, 09:44:35 pm

Author Topic: VCE Physics Question Thread!  (Read 609908 times)  Share 

0 Members and 3 Guests are viewing this topic.

mtse

  • Victorian
  • Trendsetter
  • **
  • Posts: 156
  • Quack ;)
  • Respect: +57
Re: VCE Physics Question Thread!
« Reply #1275 on: November 06, 2015, 12:44:48 pm »
+1
They aren't compulsory but you have to be reasonable with the amount of decimal places (like don't give 6 dp but don't round up so much that the value is completely different)
2014 - Math Methods CAS, Chinese SL
2015 - English, Specialist, Chemistry, Physics, Biology
2016-2020 - Monash University, Bachelor of Medicine/Bachelor of Surgery (Honours) :D
FREE UMAT WORKSHOPS     BOOK HERE!
Free VCE, UMAT + Interview Advice HERE
UMAT / Med Interview Training - contact [email protected]

Muchos Help

  • Victorian
  • Forum Regular
  • **
  • Posts: 54
  • Respect: +1
Re: VCE Physics Question Thread!
« Reply #1276 on: November 06, 2015, 11:56:04 pm »
0
Dropping in to advertise my sig.

Good luck in the physics exam boys.

Floatzel98

  • Victorian
  • Forum Leader
  • ****
  • Posts: 514
  • Respect: +16
Re: VCE Physics Question Thread!
« Reply #1277 on: November 07, 2015, 10:29:49 am »
0
Simple questions here, all from VCAA 2005 Exam 1 (Motion).

Question 5 & 7: For question 5, is this just the net force? And how do we do question 7 exactly? A bit lost on how they got B
2016 - 2019: Bachelor of Science Advanced - Research (Honours) [Physics and Mathematics] @ Monash University

jyce

  • Victorian
  • Forum Obsessive
  • ***
  • Posts: 290
  • Respect: +17
Re: VCE Physics Question Thread!
« Reply #1278 on: November 07, 2015, 11:11:51 am »
+1
Simple questions here, all from VCAA 2005 Exam 1 (Motion).

Question 5 & 7: For question 5, is this just the net force? And how do we do question 7 exactly? A bit lost on how they got B

Yes, Question 5 is just the net force and therefore is towards the centre of the circle.
For Question 7, when the train applies its breaks there are now two forces acting on the train: the force of the rails on the wheels, which acts toward the centre of the circle, and the breaking force, which acts along a tangent directly backwards from Q. On the diagram, A is the frictional force and C is the breaking force and they add together to give a force somewhere in the middle, which is B.

Floatzel98

  • Victorian
  • Forum Leader
  • ****
  • Posts: 514
  • Respect: +16
Re: VCE Physics Question Thread!
« Reply #1279 on: November 07, 2015, 11:15:50 am »
+1
Yes, Question 5 is just the net force and therefore is towards the centre of the circle.
For Question 7, when the train applies its breaks there are now two forces acting on the train: the force of the rails on the wheels, which acts toward the centre of the circle, and the breaking force, which acts along a tangent directly backwards from Q. On the diagram, A is the frictional force and C is the breaking force and they add together to give a force somewhere in the middle, which is B.
That makes sense now. Thank you!
2016 - 2019: Bachelor of Science Advanced - Research (Honours) [Physics and Mathematics] @ Monash University

Floatzel98

  • Victorian
  • Forum Leader
  • ****
  • Posts: 514
  • Respect: +16
Re: VCE Physics Question Thread!
« Reply #1280 on: November 07, 2015, 11:32:02 am »
0
Another question actually, same exam, question 13 b. I don't think I understand it fully. I haven't really seen a question like this before.  Does this angle have anything to do with the 8 degree angle it was launched at? Is it just because the ball is travelling perpendicular to the ship that it will also have the ships forward speed? I don't really understand how to explain it.

Any help would be great!
2016 - 2019: Bachelor of Science Advanced - Research (Honours) [Physics and Mathematics] @ Monash University

paper-back

  • Victorian
  • Forum Obsessive
  • ***
  • Posts: 340
  • "I must govern the clock, not be governed by it"
  • Respect: +7
Re: VCE Physics Question Thread!
« Reply #1281 on: November 07, 2015, 12:54:52 pm »
0
When a light bulb does not receive the required voltage, does it stop working entirely or does it just dim down?

jyce

  • Victorian
  • Forum Obsessive
  • ***
  • Posts: 290
  • Respect: +17
Re: VCE Physics Question Thread!
« Reply #1282 on: November 07, 2015, 01:11:12 pm »
0
When a light bulb does not receive the required voltage, does it stop working entirely or does it just dim down?

Unless it's receiving no where near the correct voltage, I believe it would just dim down. Voltage drop, after all, is a measure of how much electrical energy is being transformed. If the voltage drop across a bulb is lower than desirable, it would just be transforming less electrical energy into light, and consequently it would be shining less brightly.

Orson

  • Victorian
  • Forum Leader
  • ****
  • Posts: 776
  • Respect: +21
Re: VCE Physics Question Thread!
« Reply #1283 on: November 07, 2015, 01:20:32 pm »
0
What did you guys get for VCAA 2011 Exam 1 Question 2?

I keep getting 250N, but the answers say 1000N.

Also, what did you all get in VCAA 2013 and VCAA 2014?

Thanks!
2015: VCE
2016: BCivEng(Hons)/BCom at MU

Feel free to PM me for your engineering queries

Floatzel98

  • Victorian
  • Forum Leader
  • ****
  • Posts: 514
  • Respect: +16
Re: VCE Physics Question Thread!
« Reply #1284 on: November 07, 2015, 01:23:23 pm »
0
What did you guys get for VCAA 2011 Exam 1 Question 2?

I keep getting 250N, but the answers say 1000N.

Also, what did you all get in VCAA 2013 and VCAA 2014?

Thanks!
F = ma = 2000*0.5 = T - 0 (There are no frictional forces)
Therefore T = 1000 N

How did you get 250N exactly?
2016 - 2019: Bachelor of Science Advanced - Research (Honours) [Physics and Mathematics] @ Monash University

Orson

  • Victorian
  • Forum Leader
  • ****
  • Posts: 776
  • Respect: +21
Re: VCE Physics Question Thread!
« Reply #1285 on: November 07, 2015, 01:39:13 pm »
0
F = ma = 2000*0.5 = T - 0 (There are no frictional forces)
Therefore T = 1000 N

How did you get 250N exactly?

T = 500* 0.5 = 250N
2015: VCE
2016: BCivEng(Hons)/BCom at MU

Feel free to PM me for your engineering queries

Floatzel98

  • Victorian
  • Forum Leader
  • ****
  • Posts: 514
  • Respect: +16
Re: VCE Physics Question Thread!
« Reply #1286 on: November 07, 2015, 01:59:56 pm »
0
T = 500* 0.5 = 250N
If you wanted to find the tension through the tractor you would need to set up an equation of motion of the tractor which would need to include the driving force of it. Your equation finds the net force acting on the tractor alone.
2016 - 2019: Bachelor of Science Advanced - Research (Honours) [Physics and Mathematics] @ Monash University

dankfrank420

  • Victorian
  • Forum Leader
  • ****
  • Posts: 892
  • Respect: +52
Re: VCE Physics Question Thread!
« Reply #1287 on: November 07, 2015, 02:47:23 pm »
0
For VCAA 2014

Motion Question 2d)

It states that the "height" of the mass at the max height is 0.8m. However, I thought that considering the max extension of the spring to have a height of 0m, then the max height would therefore be 0.4m as the extension is 0.4m?

If this doesn't make sense sorry, I managed to get the entire methodology right but I just used the wrong value of h. Pretty confused tbh.


GeniDoi

  • Victorian
  • Forum Obsessive
  • ***
  • Posts: 209
  • How can mirrors be real if our i's aren't real?
  • Respect: +19
  • School: John Monash Science School
  • School Grad Year: 2015
Re: VCE Physics Question Thread!
« Reply #1288 on: November 07, 2015, 04:32:34 pm »
+1
For VCAA 2014

Motion Question 2d)

It states that the "height" of the mass at the max height is 0.8m. However, I thought that considering the max extension of the spring to have a height of 0m, then the max height would therefore be 0.4m as the extension is 0.4m?

If this doesn't make sense sorry, I managed to get the entire methodology right but I just used the wrong value of h. Pretty confused tbh.

I know the question your talking about. "Extension" refers to whatever the displacement of the end of the spring is after you've done something to it, like attaching a mass to it. If the spring was 0.4m long and you added a mass that made it's total length 1m, it's extension is 1-0.4 = 0.6m
[2014] Methods: 40
[2015 Aims] English [40+] - Specialist [40+] - Physics [43+] - Algorithmics [37?] - Psychology [32]
[2016-2019] BCommerce/BCompSci @ Monash Uni

Adequace

  • Victorian
  • Forum Obsessive
  • ***
  • Posts: 484
  • 7-1 never forget.
  • Respect: +12
Re: VCE Physics Question Thread!
« Reply #1289 on: November 07, 2015, 04:34:29 pm »
0
Might be off topic but I'm kinda getting anxious for physics next year...I looked at VCAA 2014 a couple hours ago and literally couldn't do any of it even the motion and electronics section which made me scared lmao.

I think I'm putting too many expectations on myself already and expect myself to understand U3&4 exams with my U1&2 knowledge, sigh.